NCERT Exemplar

A uniform disc of radius R, is resting on a table on its rim. The coefficient of friction between disc and table is ΞΌ. Now the disc is pulled with a force F as shown in the figure. What is the maximum value of F for which the disc rolls without slipping?

Solution: Let the linear and angular acceleration be $a$ and $\alpha$ respectively. So, $F – f = Ma$ Where, M = mass of the disc f = force of friction applied at the centre Torque to disc,...

read more

Two cylindrical hollow drums of radii R and 2R and of a common height h, are rotating with angular velocities Ο‰ (anti-clockwise) and Ο‰ (clockwise) respectively. Their axes, fixed are parallel and in a horizontal plane separated by (3R + Ξ΄). They are now brought in contact (\delta \rightarrow 0)
What would be the ratio of final angular velocities when friction ceases?

The anticlockwise and clockwise angular velocities of the drum are $\omega_{1}$ and $\omega_{2}$ respectively. When the velocities are equal, there is no force of friction and it is given as...

read more

Two cylindrical hollow drums of radii R and 2R and of a common height h, are rotating with angular velocities Ο‰ (anti-clockwise) and Ο‰ (clockwise) respectively. Their axes, fixed are parallel and in a horizontal plane separated by (3R + Ξ΄). They are now brought in contact (\delta \rightarrow 0)
a) show the frictional forces just after contact
b) identify forces and torques external to the system just after contact

a) We know, $v_{1} = \omega R$ $v_{2}=\omega^{2}R$ The direction of $v_{1}$ and $v_{2}$ are tangentially upwards in the figure, and they meet at point C. As a result, $f_{12}=-f_{21}$ represents the...

read more

A disc of radius R is rotating with an angular speed \omega_{o} about a horizontal axis. It is placed on a horizontal table. The coefficient of kinetic friction is \mu_{k}.
a) what condition should be satisfied for rolling to being?
b) calculate the time taken for the rolling to being

a) The condition that needs to be satisfied is $v_{cm}=\omega_{o}R$ b) Frictional force is responsible for allowing rolling motion to occur without the disc slipping.

read more

A disc of radius R is rotating with an angular speed \omega_{o} about a horizontal axis. It is placed on a horizontal table. The coefficient of kinetic friction is \mu_{k}.
a) what happens to the linear speed of the centre of mass when the disc is placed in contact with the table?
b) which force is responsible for the effects in previous questions?

a) The linear velocity of the revolving disc changes as it is brought into contact with the table. b) Frictional force is responsible. The figure depicts it:

read more

A disc of radius R is rotating with an angular speed \omega_{o} about a horizontal axis. It is placed on a horizontal table. The coefficient of kinetic friction is \mu_{k}.
a) what was the velocity of its centre of mass before being brought in contact with the table?
b) what happens to the linear velocity of a point on its rim when placed in contact with the table?

a) Before coming into contact with the table, the disc was rotating, and $v_{cm}=0$ was the rest. b) When the revolving disc comes into contact with the table, the linear velocity of the disc...

read more

Two discs of moments of inertia I_{1} and I_{2} about their respective axes and rotating with angular speed \omega_{1} and \omega_{2} are brought into contact face to face with their axes of rotation coincident.
a) calculate the loss in kinetic energy of the system in the process
b) account for this loss

a) Final kinetic energy = rotational + translation energy $K_{f}=KE_{R}+KE_{T}$ $\Delta \mathrm{K}=-I_{1} l_{2} / 2\left(I_{1}+l_{2}\right)\left(\omega_{1}-\omega_{2}\right) 2<0$ b) Because...

read more

Two discs of moments of inertia I_{1} and I_{2} about their respective axes and rotating with angular speed \omega_{1} and \omega_{2} are brought into contact face to face with their axes of rotation coincident.
a) does the law of conservation of angular momentum apply to the situation? why?
b) find the angular speed of the two-disc system

a) Because there is no external torque on the system and the gravitational and normal reactions to external forces have net torque zero, the equation of conservation of angular momentum can be...

read more

(n-1) equal point masses each of mass m are placed at the vertices of a regular n-polygon. The vacant vertex has a position vector concerning the centre of the polygon. Find the position vector of the centre of mass.

$r_{c m}=\frac{(n-1) m b+m a}{(n-1) m+m}$ is given Where, ${r_{cm}}$ The position of mass $m$ at the $n^{th}$ vertex is called. $r_{\mathrm{cm}}=0$ $\frac{(n-1) m b+m a}{(n-1) m+m}=0$ $(n-1) m b+m...

read more

A wheel in uniform motion about an axis passing through its centre and perpendicular to its plane is considered to be in mechanical equilibrium because no net external force or torque is required to sustain its motion. However, the particles that constitute the wheel do experience a centripetal acceleration directed towards the centre. How do you reconcile this fact with the wheel being in equilibrium? How would you set a half-wheel into uniform motion about an axis passing through the centre of mass of the wheel and perpendicular to its plane? Will you require external forces to sustain the motion?

A wheel is a stiff elastic body with a consistent motion that passes through its center, perpendicular to the wheel's plane. Due to elastic force, every particle of the wheel receives a centripetal...

read more

The vector sum of a system of non-collinear forces acting on a rigid body is given to be non-zero. If the vector sum of all the torques due to the system of forces about a certain point is found to be zero, does this mean that it is necessarily zero about any arbitrary point?

The torques' vector total equals zero. However, the net force is not zero. The following is the mathematical explanation: $G_{i} \sum_{i=1}^{n} F_{t} \neq 0$ $\tau=\tau_{1}+\tau_{2}+\ldots...

read more

Figure shows a lamina in the x-y plane. Two axes z and z’ pass perpendicular to its plane. A force F acts in the plane of lamina at point P as shown. Which of the following are true?
a) torque Ο„ caused by F about z-axis is along -\hat{k}
b) torque τ’ caused by F about z’ axis is along-\hat{k}
c) torque Ο„ caused by F about the z-axis is greater in magnitude than that about the z-axis
d) total torque is given be Ο„ = Ο„ + τ’

Correct answers are: b) torque τ’ caused by F about z’ axis is along $-\hat{k}$ c) torque Ο„ caused by F about the z-axis is greater in magnitude than that about the z-axis

read more

The net external torque on a system of particle about any axis is zero. Which of the following are compatible with it?
a) the forces may be acting radially from a point on the axis
b) the forces may be acting on the axis of rotation
c) the forces may be acting parallel to the axis of rotation
d) the torque caused by some forces may be equal and opposite to that caused by other forces

When net external torque on a system of particles about an axis is zero, torque is the cross prodeuct of $\vec r$ and $\vec F = rFsin\theta \times torque=0$ where, $\theta$ is the amgle between the...

read more

Figure shows two identical particles 1 and 2, each of mass m, moving in opposite directions with the same speed v along parallel lines. At a particular instant, r_{1} and r_{2} are their respective positions vectors drawn from point A which is in the plane of the parallel lines. Choose the correct options:
a) angular momentum l_{1} of particle 1 is about A is l_{1} =mvd_{1}
b) angular momentum l_{2} of particle 2 about A is l_{2} = mvx_{2}
c) total angular momentum of the system about A is l = mv(r_{1}+r{2})
d) total angular momentum of the system about A is l = mv(d_{2}-d_{1})

Solution: Correct answers is: d) total angular momentum of the system about A is $l = mv(d_{2}-d_{1})$ Angular momentum of particle 1 about A is given as, $\vec L_1=mvd_1$ Angular momentum of...

read more

Two long wires carrying current I_{1} and I_{2} are arranged as shown in the figure. The one carrying I1 is along is the x-axis. The other carrying current I2 is along a line parallel to the y-axis given by x = 0 and z = d. Find the force exerted at O2 because of the wire along the x-axis.

F = I(LΓ—B) = ILB sinΞΈΒ is the magnetic field B on a current-carrying wire. O2Β and I1Β Β are in the –Y direction and parallel to the y-axis I2 is perpendicular to the y-axis and parallel to the Y-axis,...

read more

Choose the correct alternatives:
a) for a general rotational motion, angular momentum L and angular velocity Ο‰ need not be parallel
b) for a rotational motion about a fixed axis, angular momentum L and angular velocity Ο‰ are always parallel
c) for a general translational motion, momentum p and velocity v is always parallel
d) for a general translational motion, acceleration a and velocity v are always parallel

a) Angular momentum L and angular velocity ω do not have to be parallel for a general rotating motion. c) Momentum p and velocity v are always parallel in a typical translational motion.

read more

On a square cardboard sheet of area

    \[784\]

    \[c{{m}^{2}}\]

, four congruent circular plates of maximum size are placed such that each circular plate touches the other two plates and each side of the square sheet is tangent to two circular plates. Find the area of the square sheet not covered by the circular plates.

Given Area of the square = \[784\] \[c{{m}^{2}}\] Hence Side of the square = \[\sqrt{Area}\] = \[\sqrt{784}\] = \[28\] cm Given that the four circular plates are congruent, Therefore diameter of...

read more

In Fig. 11.17, ABCD is a trapezium with AB || DC, AB =

    \[18\]

cm, DC =

    \[32\]

cm and distance between AB and DC =

    \[14\]

cm. If arcs of equal radii

    \[7\]

cm with centres A, B, C and D have been drawn, then find the area of the shaded region of the figure.

Solution Given AB = \[18\] cm, DC = \[32\] cm Given, Distance between AB and DC = Height = \[14\] cm We know that Β Area of the trapezium = (\[1/2\]) Γ— (Sum of parallel sides) Γ— Height =...

read more

Sides of a triangular field are

    \[15\]

m,

    \[16\]

m and

    \[17\]

m. With the three corners of the field a cow, a buffalo and a horse are tied separately with ropes of length

    \[7\]

m each to graze in the field. Find the area of the field which cannot be grazed by the three animals.

Solution From the given question, We got Sides of the triangle are \[15\] m, \[16\] m and \[17\] m. Then, perimeter of the triangle = \[(15+16+17)\] m = \[48\]m Therefore, Semi-perimeter of the...

read more

Einstein’s mass-energy relation emerging out of his famous theory of relativity relates mass (m) to energy (E) as E=mc^{2}, where c is the speed of light in vacuum. At the nuclear level, the magnitudes of energy are very small. The energy at nuclear level is usually measured in MeV where !MeV=1.6\times 10^{-13}J, the masses are measured in unified equivalent of 1u is 931.5 MeV.
a) Show that the energy equivalent of 1 u is 931.5 MeV.
b) A student writes the relation as 1 u = 931.5 MeV. The teacher points out that the relation is dimensionally incorrect. Write the correct relation.

a) The energy that is comparable to a given mass can be computed using Einstein's mass-energy relation. $1amu=1u=1.67\times 10^{-27}kg$ On Applying $E=mc^{2}$ we get, E = 931.5 MeV b) As $E=mc^{2}$...

read more

a) How many astronomical units (AU) make 1 parsec?
b) Consider the sun like a star at a distance of 2 parsec. When it is seen through a telescope with 100 magnification, what should be the angular size of the star? Sun appears to be (1/2) degree from the earth. Due to atmospheric fluctuations, eye cannot resolve objects smaller than 1 arc minute.

a) 1 parsec is the distance at which 1 AU long arc subtends an angle of 1s, according to the definition. Using the definition, we can write, 1 parsec = (3600)(180)/Ο€ AU = 206265 AU = 2 Γ— 105 AU b)...

read more

In an experiment to estimate the size of a molecule of oleic acid, 1mL of oleic acid is dissolved in 19mL of alcohol. Then 1mL of this solution is diluted to 20mL by adding alcohol. Now, 1 drop of this diluted solution is placed on water in a shallow trough. The solution spreads over the surface of water forming one molecule thick layer. Now, lycopodium powder is sprinkled evenly over the film we can calculate the thickness of the film which will give us the size of oleic acid molecule.
Read the passage carefully and answer the following questions:
a) What would be the volume of oleic acid in each mL of solution prepared?
b) How will you calculate the volume of n drops of this solution of oleic.

a) 1 mL of oleic acid is found in every 20 mL of oleic acid. This signifies that 1/20 mL of oleic acid is present in each mL of solution. Adding alcohol dilutes 1 mL of this solution to 20 mL. As a...

read more

In an experiment to estimate the size of a molecule of oleic acid, 1mL of oleic acid is dissolved in 19mL of alcohol. Then 1mL of this solution is diluted to 20mL by adding alcohol. Now, 1 drop of this diluted solution is placed on water in a shallow trough. The solution spreads over the surface of water forming one molecule thick layer. Now, lycopodium powder is sprinkled evenly over the film we can calculate the thickness of the film which will give us the size of oleic acid molecule.
Read the passage carefully and answer the following questions:
a) Why do we dissolve oleic acid in alcohol?
b) What is the role of lycopodium powder?

a) Because oleic acid does not dissolve in water, it is dissolved in alcohol. b) When oleic acid is introduced, lycopodium powder clears the circular area. This makes it possible to measure the area...

read more

An artificial satellite is revolving around a planet of mass M and radius R, in a circular orbit of radius r. From Kepler’s third law about the period of a satellite around a common central body, square of the period of revolution T is proportional to the cube of the radius of the orbit r. Show using dimensional analysis, that T = k/R √r3/g where k is a dimensionless constant and g is acceleration due to gravity.

Kepler's third law states that, $T^{2} \propto a^{3}$ i.e., square of time period $\left(T^{2}\right)$ of a satellite revolving around a planet, is proportional to the cube of the radius of the...

read more

A physical quantity X is related to four measurable quantities a, b, c and d as follows: X=a^{2}b^{3}c^{5/2}d^{-2}. The percentage error in the measurement of a, b, c and d are 1%, 2%, 3% and 4%, respectively. What is the percentage error in quantity X? If the value of X calculated on the basis of the above relation is 2.763, to what value should you round off the result.

The given physical quantity is $X=a^{2}b^{3}c^{5/2}d^{-2}$ Percentage error in X is given as (βˆ†x/x)(100) Percentage error in a is given as (βˆ†a/a)(100) = 1% Percentage error in b is given as...

read more

If Planck’s constant (h) and speed of light in vacuum (c) are taken as two fundamental quantities, which of the following can, also, be taken to express length, mass, and time in terms of the three chosen fundamental quantities?
a) mass of the electron (m_{e})
b) universal gravitational constant (G)
c) charge of the electron (e)
d) mass of proton (m_{p})

Correct answers are a) mass of electron b) universal gravitational constant and d) mass of proton

read more

Measure of two quantities along with the precision of the respective measuring instrument is:
A = 2.5 m/s Β± 0.5 m/s
B = 0.10 s Β± 0.01 s
The value of AB will be
a) (0.25 Β± 0.08) m
b) (0.25 Β± 0.5) m
c) (0.25 Β± 0.05) m
d) (0.25 Β± 0.135) m

Correct answer is a) (0.25 Β± 0.08) m Here, $\mathrm{A}=2.5 \mathrm{~ms}^{-1} \pm 0.5 \mathrm{~ms}^{-1}, \mathrm{~B}=0.10 \mathrm{~s} \pm 0.01 \mathrm{~s}$ $\mathrm{AB}=\left(2.5...

read more

Pressure versus volume graph for a real gas and an ideal gas is shown in Fig. 5.4. Answer the following questions based on this graph. (i) Interpret the behaviour of real gas with respect to an ideal gas at low pressure. (ii) Interpret the behaviour of real gas with respect to an ideal gas at high pressure. (iii)Mark the pressure and volume by drawing a line at the point where real gas behaves as an ideal gas.

(i) At low pressure as the dark blue curve and the sky blue curve are approaching each other, it shows that the real gas is behaving as an ideal gas at a low pressure. (ii) At high pressure as the...

read more

The variation of pressure with the volume of the gas at different temperatures can be graphically represented as shown in Fig. 5.3. Based on this graph answer the following questions. (i) How will the volume of a gas change if its pressure is increased at constant temperature? (ii) At constant pressure, how will the volume of a gas change if the temperature is increased from 200K to 400K?

(i) As the temperature is constant, and the pressure is increasing and the change in the volume is seen as exponentially decreasing. (ii) At constant pressure, by increasing the temperature there is...

read more

The viscosity of a liquid arises due to strong intermolecular forces existing between the molecules. Stronger the intermolecular forces, greater is the viscosity. Name the intermolecular forces existing in the following liquids and arrange them in the increasing order of their viscosities. Also, give a reason for the assigned order in one line. Water, hexane (CH3CH2CH2CH2CH2CH3), glycerine (CH2 OH CH(OH) CH2 OH)

Water has hydrogen bonding that exists as intermolecular force of attraction, hexane has Vander Waal force of attraction existing as intermolecular force of attraction, glycerin also has hydrogen...

read more

The relation between the pressure exerted by an ideal gas (Pideal) and observed pressure (Pearl) is given by the equation: Pideal = Preal+ an2/V2 If the pressure is taken in Nm-2, the number of moles in mol and volume in m3, Calculate the unit of β€˜a’. What will be the unit of β€˜a’ when pressure is in atmosphere and volume in dm3?

We know that: Pideal = Preal + an2/V2 Pideal – Preal= an2/V2 Nm-2 = a*mol2/m6 A = Nm4mol-2 The unit of β€˜a’ when the pressure is taken in Nm-2, number of moles in β€œmol” and volume in m3 is Nm4mol-2...

read more

For real gases the relation between p, V and T are given by van der Waals equation: [(P + an2) / V2](V – nb) = nRT Whereβ€˜a’ and β€˜b’ are van der Waals constants, β€˜nb’ is approximately equal to the total volume of the molecules of a gas. β€˜a’ is the measure of the magnitude of intermolecular attraction. (i) Arrange the following gases in the increasing order of β€˜b’. Give reason. O2, CO2, H2, He (ii) Arrange the following gases in the decreasing order of magnitude of β€˜a’. Give reason. CH4, O2, H2

(i) The increasing order of β€˜b’ is as follows: He < H2< O2< CO2. As the Vander Waals constant β€˜b’ is approximately equal to the total volume of the molecules of a gas. (ii)The decreasing...

read more

If the circumference of a circle and the perimeter of a square are equal, then (A) Area of the circle = Area of the square (B) Area of the circle > Area of the square (C) Area of the circle < Area of the square (D) Nothing definite can be said about the relation between the areas of the circle & square.

The correction option is (B) Area of the circle > Area of the square Explanation: From Β the given question, Circumference of a circle of radius r Β = Perimeter of a square of side a Let us take Β r...

read more

If the sum of the circumferences of two circles with radii

    \[R1\]

and

    \[R2\]

is equal to the circumference of a circle of radius

    \[R\]

, then (A)

    \[{{R}_{1}}+{{R}_{2}}=R\]

(B)

    \[{{R}_{1}}+{{R}_{2}}>R\]

(C)

    \[{{R}_{1}}+{{R}_{2}}<R\]

(D) Nothing definite can be said about the relation among

    \[{{R}_{1}}\]

,

    \[{{R}_{2}}\]

&

    \[R\]

.

The Correct option(A) \[{{R}_{1}}+{{R}_{2}}=R\] Explanation: From the given question, We got sum of the circumferences of two circles with radiiΒ \[R1\] andΒ \[R2\] is equal to the circumference of a...

read more

Two different gases β€˜A’ and β€˜B’ are filled in separate containers of equal capacity under the same conditions of temperature and pressure. On increasing the pressure slightly the gas β€˜A’ liquefies but gas B does not liquefy even on applying high pressure until it is cooled. Explain this phenomenon.

The critical temperature is the term used for this phenomenon. Here gas A liquefies means that A is below its critical temperature and gas B does not liquefy on applying high pressure as it is above...

read more

Use the information and data given below to answer the questions (a) to (c): β€’ Stronger intermolecular forces result in a higher boiling point. β€’ Strength of London forces increases with the number of electrons in the molecule. β€’ Boiling point of HF, HCl, HBr and HI is 293 K, 189 K, 206 K and 238 K respectively. (a) Which type of intermolecular forces are present in the molecules HF, HCl, HBr and HI? (b) Looking at the trend of boiling points of HCl, HBr and HI, explain out of dipole-dipole interaction and London interaction, which one is predominant here. (c) Why is the boiling point of hydrogen fluoride highest while that of hydrogen chloride lowest?

(a) Since the halides are a polar molecule (due to high electronegativity), due to the presence of permanent dipoles, the dipole-dipole interactions along with the London forces are found in HF, HCl...

read more

To construct a triangle similar to a given β–³ABC with its sides 7/3 of the corresponding sides of β–³ABC, draw a ray BX making acute angle with BC and X lies on the opposite side of A with respect to BC. The points B1, B2, …., B7 are located at equal distances on BX, B3 is joined to C and then a line segment B6Cβ€˜ is drawn parallel to B3C where Cβ€˜ lies on BC produced. Finally, line segment Aβ€˜Cβ€˜ is drawn parallel to AC.

False Support: Allow us to attempt to build the figure as given in the inquiry. Steps of development, Define a boundary section \[BC.\] With \[B\text{ }and\text{ }C\]as focuses, draw two circular...

read more

Draw a parallelogram

    \[\mathbf{ABCD}\]

in which

    \[\mathbf{BC}\text{ }=\text{ }\mathbf{5}\text{ }\mathbf{cm},\text{ }\mathbf{AB}\text{ }=\text{ }\mathbf{3}\text{ }\mathbf{cm}\]

and angle

    \[\mathbf{ABC}\text{ }=\text{ }\mathbf{60}{}^\circ ,\]

divide it into triangles

    \[\mathbf{BCD}\]

and

    \[\mathbf{ABD}\]

by the diagonal

    \[\mathbf{BD}.\]

Construct the triangle

    \[\mathbf{BD}~\mathbf{C}~\]

similar to triangle

    \[\mathbf{BDC}\]

with scale factor

    \[\mathbf{4}/\mathbf{3}\]

. Draw the line segment

    \[\mathbf{D}\mathbf{A}~\]

parallel to

    \[\mathbf{DA}\]

where

    \[\mathbf{A}\]

lies on extended side

    \[\mathbf{BA}\]

. Is

    \[\mathbf{A}\mathbf{BC}\mathbf{D}\]

a parallelogram?

Steps of construction: Β  Define a boundary \[\mathbf{AB}=\mathbf{3}\text{ }\mathbf{cm}.\] Draw a beam \[\mathbf{BY}\]making an intense \[\angle \mathbf{ABY}=\mathbf{60}{}^\circ .\] With focus...

read more

To divide a line segment

    \[\mathbf{AB}\]

in the ratio

    \[\mathbf{5}\text{ }:\text{ }\mathbf{6}\]

, draw a ray

    \[\mathbf{AX}\]

such that

    \[\angle \mathbf{BAX}\]

is an acute angle, then draw a ray

    \[\mathbf{BY}\]

parallel to

    \[\mathbf{AX}\]

and the points

    \[{{\mathbf{A}}_{\mathbf{1}}},\text{ }{{\mathbf{A}}_{\mathbf{2}}},\text{ }{{\mathbf{A}}_{\mathbf{3}}},\text{ }\ldots \]

and

    \[{{\mathbf{B}}_{\mathbf{1}}},\text{ }{{\mathbf{B}}_{\mathbf{2}}},\text{ }{{\mathbf{B}}_{\mathbf{3}}},\text{ }\ldots \]

are located at equal distances on ray

    \[\mathbf{AX}\]

and

    \[\mathbf{BY}\]

, respectively. Then the points joined are

    \[\left( \mathbf{A} \right)\text{ }{{\mathbf{A}}_{\mathbf{5}}}~\mathbf{and}\text{ }{{\mathbf{B}}_{\mathbf{6}}}~\]

    \[\left( \mathbf{B} \right)\text{ }{{\mathbf{A}}_{\mathbf{6}}}~\mathbf{and}\text{ }{{\mathbf{B}}_{\mathbf{5}}}~\]

    \[~\left( \mathbf{C} \right)\text{ }{{\mathbf{A}}_{\mathbf{4}}}~\mathbf{and}\text{ }{{\mathbf{B}}_{\mathbf{5}}}\]

    \[\left( \mathbf{D} \right)\text{ }{{\mathbf{A}}_{\mathbf{5}}}~\mathbf{and}\text{ }{{\mathbf{B}}_{\mathbf{4}}}\]

\[\left( \mathbf{A} \right)\text{ }{{\mathbf{A}}_{\mathbf{5}}}~\mathbf{and}\text{ }{{\mathbf{B}}_{\mathbf{6}}}\] As per the inquiry, A line portion \[AB\]in the proportion \[5:7\] Along these...

read more

To divide a line segment

    \[\mathbf{AB}\]

in the ratio

    \[\mathbf{5}:\mathbf{7}\]

, first a ray

    \[\mathbf{AX}\]

is drawn so that

    \[\mathbf{BAX}\]

is an acute angle and then at equal distances points are marked on the ray

    \[\mathbf{AX}\]

such that the minimum number of these points is (A)

    \[\mathbf{8}\]

(B)

    \[\mathbf{10}\]

(C)

    \[~\mathbf{11}\]

(D)

    \[\mathbf{12}\]

SOLUTION:- \[\left( D \right)\text{ }12\] As indicated by the inquiry, A line fragment \[AB\]in the proportion \[5:7\] In this way, \[A:B\text{ }=\text{ }5:7\] Presently, Draw a beam \[AX\]making an...

read more

Gases possess characteristic critical temperature which depends upon the magnitude of intermolecular forces between the particles. Following are the critical temperatures of some gases. Gases H2 He O2 N2 Critical temperature in Kelvin 33.2 5.3 154.3 126 From the above data what would be the order of liquefaction of these gases? Start writing the order from the gas liquefying first (i) H2, He, O2, N2 (ii) He, O2, H2, N2 (iii) N2, O2, He, H2 (iv) O2, N2, H2, He

The correct option is (iv) O2, N2, H2, He.

read more

Two APs have the same common difference. The first term of one AP is 2 and that of the other is 7 The difference between their 10th terms is the same as the difference between their 21st terms, which is the same as the difference between any two corresponding terms. Why?

Solution: Consider two APs with the first terms as β€˜a’ and β€˜A’. The common differencesΒ are β€˜d’ and β€˜D’, respectively. Assume that n is any term. ${{a}_{n}}~=a+\left( n-1 \right)d$ ${{A}_{n}}~=\text{...

read more

Choose the correct answer from the given four options in the following questions: The first four terms of an AP, whose first term is –2 and the common difference is –2, are
(A) – 2, 0, 2, 4 (B) – 2, 4, – 8, 16 (C) – 2, – 4, – 6, – 8 (D) – 2, – 4, – 8, –16

Solution: Option (C) – 2, – 4, – 6, – 8 is the correct answer. Explanation: First term, a = – 2 Second Term, d = – 2 ${{a}_{1\text{ }}}=~a=\text{ -}2$ It is known that the AP’s nth term is...

read more

Choose the correct answer from the given four options in the following questions: The list of numbers – 10, – 6, – 2, 2,… is (A) an AP with d = – 16 (B) an AP with d = 4 (C) an AP with d = – 4 (D) not an AP

Solution: Option (B) an AP with d = 4 is the correct answer. Explanation: According to the question, ${{a}_{1\text{ }}}=~\text{ }-10$ ${{a}_{2\text{ }}}=\text{ }-6$ ${{a}_{3\text{ }}}=~\text{ }-2$...

read more

The points A (x1, y1), B (x2, y2) and C (x3 y3) are the vertices of ABC.
(i) Find the coordinates of points Q and R on medians BE and CF, respectively such that BQ : QE = 2 : 1 and CR : RF = 2 : 1
(ii) What are the coordinates of the centroid of the triangle ABC?

Solution: (i) Let (p, q) be the coordinates of a point Q. Provided, The point Q (p, q), Divide the line joining $\mathrm{B}\left(\mathrm{x}{2}, \mathrm{y}{2}\right)$ and...

read more

The points A (x1, y1), B (x2, y2) and C (x3 y3) are the vertices of ABC. (i) The median from A meets BC at D. Find the coordinates of the point D. (ii) Find the coordinates of the point P on AD such that AP : PD = 2 : 1

Solution: According to the given question, A, B and C are the vertices of Ξ”ABC A(x1, y1), B(x2, y2), C(x3, y3) are the coordinates of A, B and C. (i) According to the information provided, D is BC's...

read more

Find the coordinates of the point Q on the x–axis which lies on the perpendicular bisector of the line segment joining the points A (–5, –2) and B(4, –2). Name the type of triangle formed by the points Q, A and B.

Solution: As the point P lies on the perpendicular bisector of AB, point Q is the midpoint of AB . By the formula for midpoint: $({{x}_{1}}~+\text{ }{{x}_{2}})/2\text{ }=\text{ }\left( -5+4...

read more

The strain volume work for an ideal gas can be determined by utilizing the articulation w= ΚƒPexdv. The work can likewise be determined from the pV–a plot by utilizing the region under the bend inside as far as possible. At the point when an ideal gas is compacted (a) reversibly or (b) irreversibly from volume Vi to Vf. pick the right alternative.

    \[\begin{array}{*{35}{l}}    \left( I \right)\text{ }w\text{ }\left( reversible \right)\text{ }=\text{ }w\text{ }\left( irreversible \right)  \\    ~  \\    \left( ii \right)\text{ }w\text{ }\left( reversible \right)\text{ }<\text{ }w\text{ }\left( irreversible \right)  \\    ~  \\    \left( iii \right)\text{ }w\text{ }\left( reversible \right)\text{ }>\text{ }w\text{ }\left( irreversible \right)  \\    ~  \\    \left( iv \right)\text{ }w\text{ }\left( reversible \right)\text{ }=\text{ }w\text{ }\left( irreversible \right)\text{ }+\text{ }pex.V  \\ \end{array}\]

Arrangement:   Alternative (ii) is the appropriate response. w (reversible) < w (irreversible) Region under the bend is more noteworthy in irreversible pressure than that of reversible...

read more